if2×4^a-3=16^a÷8^1-a
[tex].[/tex]
2×4^a-3= 16^a÷8^1-a

Answers

Answer 1

Answer:

Correct option is

B

10

Given2

2x−y

=32

⇒2

2x−y

=2

5

⇒2x−y=5−−−−(1)

Again2

x+y

=16

⇒2

x+y

=2

4

⇒x+y=4−−−−(2)

Adding(1)and(2)weget

3x=9

⇒x=3

Substitutingx=3in(2)weget

y=1.

∴x

2

+y

2

=3

2

+1

2

=10(Ans)


Related Questions

help!! this is math.

Answers

Answer:

2,78,31,64,4-2,10-6,54,-97,-38,-8-7,-4-5,-11-1,-16

What was Veronica's speed if she drove 475 miles in 6 1/4 hours?

Answers

Answer: 76 mph

Step-by-step explanation: speed is distance divided by time so 475 : 6.25 = 76 (mph)

Answer:

76 mph

Step-by-step explanation:

To find the answer, you would divide the miles she drove: 475 by the amount of time it took to get there: 6 1/4 hours. Convert the fraction to a decimal: 1/4= .25 so your problem would look like this...

475 / 6.25 = 76

In 2004 a certain oil company spent sh 1740000 on crude oil import .This was a 20%increase 0ver the money on crude oil import in 2003.
a) How much did the company spend on crude oil import in 2003.
b)What was the difference in the money spent on crude oil import in the two years?​

Answers

Answer:

a = 1,450,000

b = 290,000

Step-by-step explanation:

A: 1,740,000 = x*1.20

1,740,000/1.2 = x1.20/1.20

1,450,000 = x

B: 1,740,000 - 1,450,000 = 290,000

cos a/4= -1/4
cos a= ??

Answers

Answer:

cos a / 4 = -1/4

--->cos a =- 1          

         

                                                                 

work out the value of v°​

Answers

Answer:

v+91=180(being co-interior angle)

v=180-91

v=89,

Answer:

Step-by-step explanation:

v°+120°+91°+(180°-155°)=360°

v°+236°=360°

v°=360°-236°

v°=124°

8w-2=12w+6 both answers

Answers

Answer:

your answer is -2

I hope it's helps you

Answer:

[tex]8w - 2 = 12w + 6 \\ 8w - 12w = 6 + 2 \\ - 4w = 8 \\ w = \frac{8}{ - 4} \\ w = - 2[/tex]

I hope it helped U

stay safe stay happy

What is the slope of the line represented by the equation y = -1/2x+ 1/4​

Answers

Answer:

y = mx+ b

m=slope

-1/2

Step-by-step explanation:

Answer:

-1/2

Step-by-step explanation:

Slope-intercept form: y = mx + b

m = slopeb = y-intercept

By comparing the formula to the equation given, it is clear the slope of the line is -1/2.

What does x equal?
2x + 5x – 4= 17


Answers

Answer:

x = 3

Step-by-step explanation:

2x + 5x - 4 = 17

Combine like terms

7x - 4 = 17

Add 4 to both sides

7x = 21

Solve for x

x = 21/7

x = 3
___________________
Hope this helps!

Love you.

- doomdabomb

Answer:

x = 3

Step-by-step explanation:

2x + 5x - 4 = 17      (Given)

7x - 4 = 17              (Simplify)

7x - 4 + 4 = 17 + 4  (Add 4 on both sides)

7x = 21                   (Simplify)

7x/7 = 21/7             (Divide 7 on both sides)

x = 3                      (Simplify)

(03.03)
How does the graph of f(x) = (x – 9)^4-3 compare to the parent function g(x) =x^4?

Answers

Answer:

Step-by-step explanation:

The child graph is shifted 9 units to the right. That's what the nine does. The graph also shifts 3 units down when compared to g(x).

I have put a graph in so that you can see the shifts for yourself.

The red line is g(x) = x^4

The blue line is f(x) = (x - 9)^4 - 3

m−4=2m what does m= Please help

Answers

Answer:

-4

Step-by-step explanation:

you would subtract m from both sides to get -4=m

(b) Eric removed 72 fish from his pond over a period of 9 days. He removed the same number of
fish each day, What was the change in the number of fish in the pond each day?
fish



FASTEST WIL BE THE BRAINLIEST

Answers

Answer:

-8

Step-by-step explanation:

72 ÷ 9 = 8

The fish are being taken out, so negative.

I hope this helps!

pls ❤ and mark brainliest pls!

Find a polynomial f(x) of degree 4 that has the following zeros. 2, 5, 0, -7

Answers

Answer:

Step-by-step explanation:

f(x) = (x - 2)(x - 5)x(x+ 7)

f(x) = (x^2 - 7x + 10)*x * (x + 7)

f(x) = x(x^3 - 39x + 70)

f(x) = x^4 - 39x^2 + 70x

To show that this is correct, I've made a graph with these points labeled. The graph is just around the x axis. The local maximums and minimums are just too large a value.

Write a function C(t) to represent the cost of
renting a canoe for t hours. The rental charge
is $25 plus $2 for every 20 minutes of rental.
Find C(1.5) and describe its meaning.

Answers

Answer:

C(t)=25+2t

C(1.5)=25+2(1.5)=28

$28 for 1.5 minutes rent

A function C(t) represents the cost of renting a canoe for t hours is C(t)=25+2t and the cost for 1.5 minutes will be $28.

What is a function?

A function is defined as the expression that set up the relationship between the dependent variable and independent variable. A function in mathematics from a set X to a set Y allocates exactly one element of Y to each element of X. The sets X and Y are collectively referred to as the function's domain and codomain, respectively.

Given that the rental charge is $25 plus $2 for every 20 minutes of rental.

The function formed for the given data will be:-

C(t)=25+2t

At 1.5 minutes the rent will be calculated as below:-

C(1.5)=25+2(1.5)=28

Therefore, the function C(t) represents the cost of renting a canoe for t hours is C(t)=25+2t, and the cost for 1.5 minutes will be $28.

To know more about the function follow

https://brainly.com/question/1942755

#SPJ2

Emma jogs 2 miles along the beach in 1/3 of an hour. If she travels at a constant rate, how far will she jog in an hour?

Answers

Answer:

6

Step-by-step explanation:

you do 2 times 3 because you 2 miles is only 1/3 and you want do get the answer for an hour so 2/3 is 6

Answer:

6 miles

Step-by-step explanation:

If emma jogs at a constant rate she will travel 6 miles in 1 hour.

1/3 of an hour times 3 equals 1 hour and the 2 miles every 1/3 times 3 will be 6

hope it helps

why are there three zeros in the product of 6 times 103

Answers

Answer:

There’s not

Step-by-step explanation:

It’s 618

simplify each expression

Answers

Answer:

[tex] {x}^{ \frac{1}{2} } \times {x}^{ \frac{2}{3} } [/tex]

[tex] {x}^{ \frac{3 + 4}{6} } [/tex]

[tex] {x}^{ \frac{7}{6} } [/tex]

Part A Which features describe the graph? Select all that apply. A domain: (-3, 4] B. range: (-3, 3] o C. increasing: (-3. -1) D. decreasing: (-1, 4) E positive: (-2, 2) F. negative: (-3.-2). (2, 4] Part B What is the average rate of change over the interval [-2. O for the graph in Part A? average rate of change =

Answers

Answer:

C, F

Step-by-step explanation:

The quotient of 31 and the product of a number and - 8?

Answers

Answer:

[tex]\frac{31}{-8n}[/tex]

Step-by-step explanation:

Overall, the expression is a quotient, and you're dividing 31 by whatever the product is of -8 and an unknown number (a variable). The product of -8 and a number means you multiply them together

HELP ASAP!! THIS IS SLOPE INTERCEPT FORM AND I HAVE NO IDEA HOW TO TO DO PLEASE EXPLAIN

Answers

Heya!

Wassup?

See the above pictures!

Remember: Make a dot where the line touches the graph.

Hope it help you

Which expression has a value of 13?


(12+6)÷3+3

12+6÷(3+3)

12+(6÷3+3)

(12+6)÷(3+3)

Answers

Answer:

Option b 12+6÷(3+3)

Step-by-step explanation:

12+6÷(3+3)

= 12+6÷6

= 12+1

= 13

A man is paid $1500 salary. He spends 20% of the salary for his kids education, 35% for food, 15% for miscellaneous and he saves the rest. How much money does he save?​

Answers

Answer:

450

Step-by-step explanation:

The total is 100 %

20% + 35% +15% = 70%

100-70 = 30

He saves 30%

1500 * 30%

1500 *.30

450

G I V E N :

A man is paid $1500 salary. He spends 20% of the salary for his kids education, 35% for food, 15% for miscellaneous and he saves the rest. How much money does he save?

S O L U T I O N :

Total Income = $ 1500

According to the question,

He spends 20% to his kids education

He spends 35% for food

He spends 15% for miscellaneous

Out of total income he spends 20% on kids education

So, amount spent on kids education = 20% of $1500

Amount = 20/100 × $1500

Amount = $300

Hence, money spent on his kids education is $300

35% is spent on food

Money spent on food = 35% of $1500

Money spent = 35/100 × $1500

Money spent = $525

Hence, money spent on food is $525

15% is spent on miscellaneous

Money spent on miscellaneous = 15% of $1500

Money spent = 15/100 × $1500

Money spent = $225

Hence, money spent on miscellaneous is $225

Now, adding all the three we get

$300 + $525 + $225

$825 + $225

$1050

Total money spent = $1050

Remaining money = $1500 - $1050 = $450

Remaining amount left with him is $450

The fifty people at a library are only allowed to sit in groups of 3,4, or 5 people. If all 50 people are seated, the greatest possible number of groups is

Answers

Answer:

16

Step-by-step explanation:

50÷3=16R2

Second largest multiple of 3 that is smaller than 50=45

45÷3=15 groups of 3 people each

50-45=5

         =1 group of 5 people

Greatest possible number of groups=15+1

                                                            =16

A company that makes barber scissors has found that the sales volume for barber scissors is directly proportional to the size of the sales force and inversely proportional to the price of the scissors. If 19440 pairs were sold in 2003, when there were 54 salespeople and the price was $20 a pair, what is the sales forecast for the upcoming year with 50 salespeople and a price of $18 a pair?

Answers

Answer:

20,000 pairs of barber scissors.

Step-by-step explanation:

The sales volume V for barber scissors is directly proportional to the size S of the sales force and inversely proportional to the price P of the scissors.

In other words, we can write that:

[tex]\displaystyle V = \frac{kS}{P}[/tex]

Where k is the constant of proportionality.

19,440 pairs were sold in 2013, when there was 54 salespeople and the price was $20 a pair. Hence, V = 19440 when S = 54 and P = 20. Substitute and determine the value of k:

[tex]\displaystyle \begin{aligned} (19440) &= \frac{k(54)}{(20)} \\ \\ 19440 &= \frac{27}{10}k \\ \\ k &=\frac{10}{27}(19440) = 7200 \end{aligned}[/tex]

Hence, our equation is:

[tex]\displaystyle V = \frac{7200S}{P}[/tex]

We want to determine the sales forecast with 50 salespeople and a price of $18 a pair. Hence, S = 50 and P = 18:

[tex]\displaystyle \begin{aligned} V&= \frac{7200(50)}{(18)} \\ \\ &= 20000\end{aligned}[/tex]

In conclusion, 20,000 pairs of barber scissors can be expected to be sold.

Solve the following complex equation

18+22i=3x-2yi

Answers

Answer:

[tex]this \: a \: complex \: equation \: so \: compare \\ \: real \: term \: and \: imaginary \: term \\ so \\ 18 = 3x \: \: and \: 22i = - 2yi \\ x = \frac{18}{3} \: \: \: \: and \: y = \frac{22i}{ - 2i} \\ x = 6 \: \: \: y = - 11 \\ then \: 6 \times 3 - - 2 \times 11i \\ = 18 + 22i \\ thank \: you[/tex]

Simplify root 20 divided by root 5​

Answers

Answer:

2

Step-by-step explanation:

√20 ÷ √5

√20/√5

=> √(20/5)

=> √4

=> 2

The algebraic expression for The quotient of 86 and x

Answers

Answer: 86 ÷ x or 86 / x

Step-by-step explanation:

A quotient is a number that results when one number is divided by another.

Therefore, in order to get the quotient, numbers or terms should process division which the final answer would be quotient.

In the process, [÷ ] or [/] the division signs shall be applied.

The quotient of 86 and x represents 86 divided by x, which the final result would be [tex]\boxed {86/x}[/tex]

Hope this helps!! :)

Please let me know if you have any questions

I believe the answer you are looking for is simply just 86÷x or 86/x.

if f(X+4) =3x+4 , then find the value of f(4)
please explain it clearly .​

Answers

[tex]\\ \sf\longmapsto f(x+4)=3x+4[/tex]

[tex]\\ \sf\longmapsto f(x)=3x+4-4[/tex]

[tex]\\ \sf\longmapsto f(x)=3x[/tex]

Now

[tex]\\ \sf\longmapsto f(4)[/tex]

[tex]\\ \sf\longmapsto 3(4)[/tex]

[tex]\\ \sf\longmapsto 12[/tex]

3/5 of the T-shirts and a T-shirt shop are blue 5/8 of those T-shirts are on sale 1/3 of the blue T-shirts that are on sale or size medium what fraction of the shops T-shirts are blue T-shirts that are on sale and are size medium

Answers

Answer:

yes their size is medium

WILL GIVE BRAINLIEST TO WHOEVER CAN EXPLAIN:
True or False-in scientific notation when you move the decimal point left is it positive.

Answers

Answer:

false.

Step-by-step explanation:

just cause you move a decimal point to the left dosent mean its positive.

Pls help on the top question

Answers

Answer:

-13=5×1+5×4m-2m

-13=5+20m-2m

-13-5=18m

-18=18m

18m=-18

m=18÷18

m=1

Answer:

[tex]\frac{55}{4}[/tex]

Step-by-step explanation:

let the number be n the multiply by - 4 is - 4n and add 5 gives

- 4n + 5 = - 50 ( subtract 5 from both sides )

- 4n = - 55 ( divide both sides by - 4 )

n = [tex]\frac{-55}{-4}[/tex] = [tex]\frac{55}{4}[/tex]

The number is [tex]\frac{55}{4}[/tex] or 13.75 if you prefer

Other Questions
Select the letter of the correct answer.The article talks mainly aboutA. unusual marketing stunts that have been done inspaceB. different ways that companies market their productsC. different ways that companies get customers to helpthemD. unusual marketing stunts that have gone wrong Indicate which is the largest decimal in each of the following pairs.0.0006 and 0.006 Where did the first Americans come from? Where did they migrate? A design for a banner includes a striped background. The stripes are colored in the repeating order: yellow, blue, orange, green, and purple. What calor is the 37th stripe? The function of a tendon is:A. Binds muscle to boneB. Binds bone to boneC. Where two bones meetD. Causes flexion Evaluate (3.4 x 10^)(4.5 x 103). Write youranswer in scientific notation. 6x + 3y - 9 whenx = 6, y = 9 A ball is thrown into the air with an upward velocity of 80 feet per second. The functionh = -16t2 + 80t models the height h, in feet, of the ball at time t, in seconds. When will the ball reach the ground?after 3 secondsafter 4 secondsafter 5 secondsafter 6 secondsBest explanation gets Brainliest!! nWho do lobbyists try to influencein an attempt to sway a vote?A. members of CongressB. the presidentC. Supreme Court judges please helpanswer choices A. (-5, 2)B. (5, 2)C. (2, -5)D (-2, 5) Textual evidence that supports why Guy Montag is an Anti-heroQuote. Page number it confuses me when i have to fill in the in between i just dont quite understand how to work it out please explain it clearly Omg some one please help conflicts in wuthering heights(internal and external ) Help me Guess just help please 9x + 69 = 70. Find the value of x What is the summer sleep of frog called?? what is the arithmetic sequence of a1=228 n=28 sn=2982 Hacer una lista, en espaol, con el nombre de todos los pases que hablan el espaol como primeralengua y sus respectivas capitales. 1. Which of the following is a true statement?a. To square a number, multiply the numberby 2.b. The inverse of squaring a number is todivide the number by 2.c. To square a number, multiply the numberby itself.d. A perfect square is a number whosesquare root is an even number. SHOW PROOF